You are on page 1of 24

Solved Paper 2013

Joint Entrance Examination


Time : 3 hrs

MM : 360

Instructions
1.

This test consists of 90 questions.

2.

There are three parts in the question paper A, B, C consisting of Physics, Chemistry and Mathematics
having 30 questions in each part of equal weightage. Each question is allotted 4 marks for correct
response.

3.

Candidates will be awarded marks as stated above for correct response of each question. 1/4 mark will
be deducted for indicating incorrect response of each question. No deduction from the total score will
be made if no response is indicated for an item in the answer sheet.

4.

There is only one correct response for each question. Filling up more than one response in any
question will be treated as wrong response and marks for wrong response will be deducted according
as per instructions.

Physics
1. A uniform cylinder of length L and mass
M having cross-sectional area A is
suspended, with its length vertical from a
fixed point by a massless spring such that
it is half submerged in a liquid of density
at equilibrium position. The extension x0
of the spring when it is in equilibrium is
Mg
k
Mg
(b)
k
Mg
(c)
k
Mg
(d)
k
(a)

1 LA

M
LA

2M
1 + LA

2. A metallic rod of

2l

length l is tied to a

string of length 2l
and made to rotate
with angular speed
on a horizontal
table with one end of the string fixed. If
there is a vertical magnetic field B in the
region, the emf induced across the ends of
the rod is
2B l3
2
4B l 2
(c)
2

(a)

3B l 3
2
5B l 2
(d)
2
(b)

JEE Main Solved Paper 2013

27

3. This question has Statement I and

7. Two capacitors C1 and C2 are charged to 120 V

Statement II. Of the four choices


given after the statements, choose the
one that best describes the two
statements.
Statement I A point particle of mass
m moving with speed v collides with
stationary point particle of mass M. If
the maximum energy loss possible is
1
m
given as f mv 2 , then f =
.

2
M + m

and 200 V respectively. It is found that by


connecting them together the potential on
each one can be made zero. Then,

Statement II Maximum energy loss


occurs when the particles get stuck
together as a result of the collision.
(a) Statement I is true, Statement II is true;
Statement II is the correct explanation of
Statement I
(b) Statement I is true, Statement II is true;
Statement II is not the correct explanation
of Statement I
(c) Statement I is true, Statement II is false
(d) Statement I is false, Statement II is true

4. Let [ 0 ] denotes the dimensional


formula of the permittivity of vacuum.
If M = mass, L = length, T = time and
A = electric current, then
(a) [0 ] = [M1 L3 T2 A ]

(b) [0 ] = [M1 L3 T4 A 2 ]

(c) [0 ] = [M2 L2 T1 A 2 ]

8. A sonometer wire of length 1.5 m is made of


steel. The tension in it produces an elastic
strain of 1%. What is the fundamental
frequency of steel, if density and elasticity of
steel are 7.7 103 kg/m3 and 2.2 1011 N /m 2
respectively?
(a) 188.5 Hz
(c) 200.5 Hz

(b) 178.2 Hz
(d) 770 Hz

9. A circular loop of radius 0.3 cm lies parallel to


a much bigger circular loop of radius 20 cm.
The centre of the smaller loop is on the axis of
the bigger loop. The distance between their
centres is 15 cm. If a current of 2.0 A flows
through the smaller loop, then the flux linked
with bigger loop is
(a) 9.1 1011 Wb
11

(c) 3.3 10

Wb

(b) 6 1011 Wb
(d) 6.6 109 Wb

10. Diameter of a plano-convex lens is 6 cm and


thickness at the centre is 3 mm. If speed of
light in material of lens is 2 108 m/s, the
focal length of the lens is

11. What is the minimum energy required to

5. A projectile is given an initial velocity

of ( $i + 2 $j) m/s, where $i is along the


ground and $j is along the vertical. If
g = 10 m/s2 , the equation of its
trajectory is
(a) y = x 5 x2

(b) y = 2 x 5 x2
(c) 4 y = 2 x 5 x2

launch a satellite of mass m from the surface


of a planet of mass M and radius R in a
circular orbit at an altitude of 2R?
5GmM
6R
GmM
(c)
2R
(a)

2 GmM
3R
GmM
(d)
3R
(b)

12. A diode detector is used to detect an

(d) 4 y = 2 x 25 x2

6. The amplitude of a damped oscillator


decreases to 0.9 times its original
magnitude is 5 s. In another 10 s, it
will decrease to times its original
magnitude, where equals
(b) 0.81

(b) 3C1 = 5C 2
(d) 9C1 = 4C 2

(a) 15 cm (b) 20 cm (c) 30 cm (d) 10 cm

(d) [0 ] = [M1 L2 T1 A 2 ]

(a) 0.7

(a) 5C1 = 3C 2
(c) 3C1 + 5C 2 = 0

(c) 0.729

(d) 0.6

amplitude
modulated
wave
of 60%
modulation by using a condenser of capacity
250 pico farad in parallel with a load
resistance 100 k. Find the maximum
modulated frequency which could be
detected by it.
(a) 10.62 MHz
(c) 5.31 MHz

(b) 10.62 kHz


(d) 5.31 kHz

28 JEE Main Solved Papers


13. A beam of unpolarised light of intensity I0
is passed through a polaroid A and then
through another polaroid B which is
oriented so that its principal plane makes
an angle of 45 relative to that of A. The
intensity of the emergent light is
(b) I0 /2
(d) I0 / 8

(a) I0
(c) I0 / 4

14. The supply voltage to room is 120 V. The

resistance of the lead wires is 6 . A 60 W


bulb is already switched on. What is the
decrease of voltage across the bulb, when
a 240 W heater is switched on in parallel
to the bulb?
(a) Zero
(c) 13.3 V

(b) 2.9 V
(d) 10.04 V

15. The shown p-V diagram represents the


thermodynamic cycle of an engine,
operating with an ideal monoatomic gas.
The amount of heat, extracted from the
source in a single cycle is

the cylinder have equal cross-sectional


area A. When the piston is in equilibrium,
the volume of the gas is V0 and its pressure
is p0 . The piston is slightly displaced from
the equilibrium position and released.
Assuming that the system is completely
isolated from its surrounding, the piston
executes a simple harmonic motion with
frequency
(a)

1 A p0
2 V0 M

(c)

1
2

p0

C
D

2V0

(a) p0 V0

13
(b) p0 V0
2

11
(c) p0 V0
2

(d) 4 p0 V0

16. A hoop of radius r and mass m rotating

with an angular velocity 0 is placed on a


rough horizontal surface. The initial
velocity of the centre of the hoop is zero.
What will be the velocity of the centre of
the hoop when it ceases to slip?
r0
4
r0
(c)
2

(a)

(b)

r0
3

(d) r0

17. An ideal gas enclosed in a vertical


cylindrical container supports a freely
moving piston of mass M. The piston and

(d)

1
2

MV0
A p0

temperature and then allowed to cool in


a room which is at temperature 0 . The
graph between the temperature T of the
metal and time t will be closed to
T

(a) T

(b)
0
t

(c)
0

(d)
0

V0

1 V0 Mp0
2 A2

18. If a piece of metal is heated to

2p0

A 2 p0
MV0

(b)

19. This question has Statement I and


Statement II. Of the four choices given
after the statements, choose the one that
best describes the two statements.
Statement I Higher the range, greater is
the resistance of ammeter.
Statement II To increase the range of
ammeter, additional shunt needs to be
used across it.
(a) Statement I is true, Statement II is true;
Statement II is the correct explanation of
Statement I
(b) Statement I is true, Statement II is true;
Statement II is not the correct explanation of
Statement I
(c) Statement I is true, Statement II is false
(d) Statement I is false, Statement II is true

JEE Main Solved Paper 2013


20. In an L-C-R circuit

24. The I-V characteristic of an LED is

as shown, both
S1
switches are open R
C
initially.
Now,
switch S1 and S2 ,
S2
L
kept open. (q is
charge on the
capacitor and = RC is capacitance time
constant). Which of the following
statement is correct?
(a) Work done by the battery is half of the energy
dissipated in the resistor
(b) At t = , q = CV /2
(c) At t = 2, q = CV (1 e 2 )

(d) At t = , q = CV (1 e 1 )
2

21. Two coherent point


sources S1 and S2 are
separated by a small
distance d as shown.
The fringes obtained
on the screen will be
(a) points
(c) semi-circle

Screen
d
S1 S2
D

(b) straight lines


(d) concentric circles

22. The magnetic field in a travelling


electromagnetic wave has a peak value of
20 nT. The peak value of electric field
strength is
(a) 3 V / m
(c) 9 V / m

(b) 6 V / m
(d) 12 V / m

23. The anode voltage of a photocells kept


fixed. The wavelength of the light
falling on the cathode is gradually
changed. The plate current I of photocell
varies as follows
I

R YG B

B
G

(a) I

(b) Y

V
V

(c) I

(d)

R
Y
G
B

by decrease in its surface energy, so that


its temperature remains unchanged.
What should be the minimum radius of
the drop for this to be possible? The
surface tension is T , denstiy of liquid is
and L is its latent heat of vapourisation
(a) L /T
(c) T /L

(b) T /L
(d) 2 T /L

26. In a hydrogen like atom, electron makes


transition from an energy level with
quantum number n to another with
quantum number ( n 1.
) If n >> 1, the
frequency of radiation emitted is
proportional to
(a)

1
n

(c)

1
n3 / 2

1
n2
1
(d) 3
n
(b)

27. The graph between angle of deviation ()


and angle of incidence (i) for a triangular
prism is represented by

(a)

(b)

(b)
O

(c)

(c)

(d)

O
- Red
- Yellow
- Green
- Blue

25. Assume that a drop of liquid evaporates

(a)

29

(d)
O

30 JEE Main Solved Papers


28. Two charges, each equal to q, are kept at
x = a and x = a on the X-axis. A particle
q
of mass m and charge q0 = is placed at
2
the origin. If charge q0 is given a small
displacement( y << a) along the Y-axis, the
net force acting on the particle is
proportional to
(b) y

(a) y

(c)

1
y

(d)

1
y

29. Two short bar magnets of length 1 cm

each have magnetic moments 1.20 Am 2


and 1.00 Am 2 respectively. They are
placed on a horizontal table parallel to
each other with their N poles pointing
towards the south. They have a common
magnetic equator and are separated by a
distance of 20.0 cm. The value of the
resultant horizontal magnetic induction

at the mid-point O of the line joining their


centres is close to (horizontal component
of the earths magnetic induction is
3.6 10 5 Wb/m 2 )
(a) 3.6 105 Wb / m2 (b) 2.56 104 Wb / m2
(c) 3.50 104 Wb / m2 (d) 5.80 104 Wb / m2

30. A charge Q is uniformly distributed over a


long rod AB of length L as shown in the
figure. The electric potential at the point
O lying at distance L from the end A is
A

(a)

Q
8 0 L

(b)

3Q
4 0 L

(c)

Q
4 0 L ln 2

(d)

Q ln 2
4 0 L

Chemistry
31. Which of the following complex species is

35. A piston filled with 0.04 mole of an ideal

not expected to exhibit optical isomerism?

gas expands reversibly from 50.0 mL to


375 mL at a constant temperature of
37.0C. As it does so, it absorbs 208 J of
heat. The values of q and W for the
process will be
(R = 8.314 J / molK, ln 7.5 =2.01)

(a) [Co(en)3 ]3+


(c) [Co (NH3 )3Cl 3 ]

(b) [Co(en)2 Cl 2 ]+

(d) [Co(en)(NH3 )2 Cl 2 ]+

32. Which one of the following molecules is


expected
to
behaviour?
(a) C 2
(c) O 2

exhibit

diamagnetic

(b) N 2
(d) S 2

33. A solution of ( )-1-chloro-1-phenylethane


in toluene racemises slowly in the
presence of a small amount of SbCl5 , due
to the formation of
(a) carbanion
(c) carbocation

(b) carbene
(d) free radical

3+
34. Given, ECr
= 0.74 V;
/Cr
/Mn 2+ = 1.51 V
EMnO
4

ECr

O 2 /Cr 3+
2
7

= 1.36 V
= 1.33 V; ECl
/Cl

Based on the data given above, strongest


oxidising agent will be
(a) Cl
(c) Mn 2+

(b) Cr 3+
(d) MnO 4

(a)
(b)
(c)
(d)

q
q
q
q

= + 208 J, W
= 208 J, W
= 208 J, W
= + 208 J, W

= 208 J
= 208 J
= + 208 J
= + 208 J

36. The molarity of a solution obtained by


mixing 750 mL of 0.5 (M) HCl with
250 mL of 2(M) HCl will be
(a) 0.875 M
(c) 1.75 M

(b) 1.00 M
(d) 0.0975 M

37. Arrange the following compounds in the


order of decreasing acidity
OH
OH
OH
;

Cl

CH3

(I)

(II)

(a) II > IV > I > III


(c) III > I > II > IV

OH
;

NO2
(III)

OCH3
(IV)

(b) I > II > III > IV


(d) IV > III > I > II

JEE Main Solved Paper 2013


38. For gaseous state, if most probable speed
is denoted by C *, average speed by C and
mean square speed by C, then for a large
number of molecules, the ratios of these
speeds are
(a)
(b)
(c)
(d)

C * : C : C = 1.225 : 1.128 : 1
C * : C : C = 1.128 : 1.225 : 1
C * : C : C = 1 : 1.128 : 1.225
C * : C : C = 1 : 1.225 : 1.128

39. The rate of a reaction double when its


temperature changes from 300 K to 310 K.
Activation energy of such a reaction will
be ( R = 8.314 JK 1 mol1 and log 2 = 0.301)
(a) 53.6 kJ mol 1
(c) 58.5 kJ mol 1

(b) 48.6 kJ mol 1


(d) 60.5 kJ mol 1

40. A compound with molecular mass 180 is


acylated with CH3 COCl to get a
compound with molecular mass 390. The
number of amino groups present per
molecule of the former compound is
(a) 2

(b) 5

(c) 4

41. Which of the following arrangements


does not represent the correct order of the
property stated against it?
(a) V 2+ < Cr 2+ < Mn2+ < Fe 2+ :
paramagnetic
behaviour
(b) Ni 2+ < Co 2+ < Fe 2+ < Mn2+ : ionic size
(c) Co 3+ < Fe 3+ < Cr 3+ < Sc 3+ : stability in
aqueous solution
(d) Sc < Ti < Cr < Mn : number of oxidation
states

42. The order of stability of the following


carbocations

CH

CH2; CH3

(I)

CH2

CH2;

is

(II)
(III)

(a) III > II > I


(c) I > II > III

(a) 5, 2 and 16
(c) 2, 5 and 16

(b) 2, 5 and 8
(d) 5, 2 and 8

44. Which of the following is the wrong


statement?
(a)
(b)
(c)
(d)

ONCl and ONO are not isoelectronic


O 3 molecule is bent
Ozone is violet-black in solid state
Ozone is diamagnetic gas

45. A gaseous hydrocarbon gives upon


combustion 0.72 g of water and 3.08 g of
CO2 . The empirical formula of the
hydrocarbon is
(a) C 2H4
(c) C 6H5

(b) C 3H4
(d) C 7 H8

46. In which of the following pairs of


molecules/ions, both the species are not
likely to exist?
(a) H+2 , He 2
2

(b) H 2 , He 22
(d) H2 , He 22+

47. Which of the following exists as covalent


crystals in the solid state?
(a) Iodine
(c) Sulphur

(b) Silicon
(d) Phosphorus

48. Synthesis of each molecule of glucose in


photosynthesis involves
(a) 18 molecules of ATP
(b) 10 molecules of ATP
(c) 8 molecules of ATP
(d) 6 molecules of ATP

49. The coagulating power of electrolytes


CH2

CH2

The values of x , y and z in the reaction are


respectively

(c) H22 + , He 2

(d) 6

31

(b) II > III > I


(d) III > I > II

(a)
(b)
(c)
(d)

Al 3+ < Ba 2 + < Na +
Na + < Ba 2+ < Al 3+
Ba 2+ < Na 2+ < Al 3+
Al 3+ < Na + < Ba 2+

50. Which of the following represents the


correct order of increasing first ionisation
enthalpy for Ca, Ba, S, Se and Ar?

43. Consider the following reaction,


xMnO4 + yC2O24 + zH+ xMn2+
+ 2yCO2 +

having ions
Na + , Al3 + and Ba2 + for
arsenic sulphide sol increases in the order

z
HO
2 2

(a)
(b)
(c)
(d)

Ca < S < Ba < Se < Ar


S < Se < Ca < Ba < Ar
Ba < Ca < Se < S < Ar
Ca < Ba < S < Se < Ar

32 JEE Main Solved Papers


51. Energy of an electron is given by
E = 2.178 10 18

55. The first ionisation potential of Na is


5.1 eV. The value of electron gain
enthalpy of Na+ will be

Z2
J 2
n

Wavelength of light required to excite an


electron in an hydrogen atom from level
n = 1 to n = 2 will be (h = 6.62 103 4 Js
and c = 3.0 108 ms1)
7

(a) 1.214 10

(c) 6.500 10

m
m

(b) 2.816 10

(d) 8.500 10

m
m

52. Compound ( A), C8 H9 Br gives a white


precipitate when warmed with alcoholic
AgNO3 . Oxidation of (A) gives an acid
(B), C8 H6 O4 . (B) easily forms anhydride
on heating. Identify the compound (A).
CH2Br
(a)

C2H5
(b)
Br

CH3
CH2Br
(c)

(d)

CH2Br
CH3

CH3

53. Four successive members of the first row


transition elements listed below with
atomic numbers. Which one of them is

expected to have the highest EM


3+
/ M 2+
value?
(a) Cr (Z = 24)
(c) Fe (Z = 26)

(b) Mn (Z = 25)
(d) Co (Z = 27)

54. How many litres of water must be added


to 1 L of an aqueous solution of HCl with a
pH of 1 to create an aqueous solution with
pH of 2?
(a) 0.1 L

(b) 0.9 L

(c) 2.0 L

(d) 9.0 L

(a) 2.55 eV

(b) 5.1 eV

(c) 10.2 eV

(d) + 2.55 eV

56. An organic compound A upon reacting


with NH 3 gives B. On heating, B gives C. C
in the presence of KOH reacts with Br2 to
give CH3CH2NH2 . A is
(a) CH3COOH
(b) CH3CH2CH2COOH
(c) CH3 CH COOH (d) CH3CH2COOH

CH3

57. Stability of the species Li2 , Li2 and Li2+


increases in the order of
(a) Li 2 < Li +2 < Li 2
(c) Li 2 < Li 2 < Li +2

(b) Li 2 < Li +2 < Li 2


(d) Li 2 < Li 2 < Li 2+

58. An unknown alcohol is treated with the


Lucas reagent to determine whether the
alcohol is primary, secondary or tertiary.
Which alcohol reacts fastest and by what
mechanism?
(a) Secondary alcohol by SN1
(b) Tertiary alcohol by SN1
(c) Secondary alcohol by SN2
(d) Tertiary alcohol by SN2

59. The gas leaked from a storage tank of the


Union Carbide plant in Bhopal gas
tragedy was
(a) Methylisocyanate
(c) Ammonia

(b) Methylamine
(d) Phosgene

60. Experimentally it was found that a metal


oxide has formula M0.98O. Metal M,
present as M 2+ and M 3+ in its oxide.
Fraction of the metal which exists as M3 +
would be
(a) 7.01%
(c) 6.05%

(b) 4.08%
(d) 5.08%

JEE Main Solved Paper 2013

33

Mathematics
61. Distance between two parallel planes

67. The sum of first 20 terms of the sequence

2 x + y + 2 z = 8 and 4 x + 2 y + 4 z + 5 = 0 is

0.7, 0.77, 0.777,, is

3
(a)
2

(a)

5
(b)
2

7
(c)
2

9
(d)
2

62. At present, a firm is manufacturing


2000 items. It is estimated that the rate of
change of production P with respect to
additional number of workers x is given
dP
by
= 100 12 x . If the firm employees
dx
25 more workers, then the new level of
production of items is
(a) 2500
(c) 3500

(b) 3000
(d) 4500

63. Let A and B be two sets containing


2 elements and 4 elements respectively.
The number of subsets of A B having 3
or more elements is
(a) 256

(b) 220

(c) 219

(d) 211

64. If the lines

x 2 y 3 z 4
x 1 y 4 z 5
and
=
=
=
=
1
1
k
k
2
1
are coplanar, then k can have
(a)
(b)
(c)
(d)

any value
exactly one value
exactly two values
exactly three values
^

the
vectors
AB = 3 i$ + 4 k and
$
$
$
AC = 5 i 2 j + 4 k are the sides of a ABC,
then the length of the median through A
is
18

(b)

72

(c)

33

(d)

45

66. The real number k for which the equation,


2 x 3 + 3 x + k = 0 has two distinct real roots
in [0, 1]
(a)
(b)
(c)
(d)

68. A ray of light along x + 3 y = 3 gets


reflected upon reaching X-axis, the
equation of the reflected ray is
(a) y = x + 3
(b) 3 y = x 3
(c) y = 3 x 3
(d) 3 y = x 1

69. The number of values of k, for which the


system of equations
(k + 1)x + 8y = 4k
kx + (k + 3)y = 3k 1
has no solution, is
(a) infinite
(c) 2

(b) 1
(d) 3

70. If the equations x 2 + 2 x + 3 = 0 and


ax 2 + bx + c = 0,
a, b, c R,
common root, then a : b : c is

65. If

(a)

7
(179 10 20 )
81
7
(b) (99 10 20 )
9
7
(c)
(179 + 10 20 )
81
7
(d) (99 + 10 20 )
9

lies between 1 and 2


lies between 2 and 3
lies between 1and 0
does not exist

(a) 1 : 2 : 3
(c) 1 : 3 : 2

have

(b) 3 : 2 : 1
(d) 3 : 1 : 2

71. The circle passing through (1, 2) and


touching the axis of x at (3, 0) also passes
through the point
(a) ( 5, 2 )
(c) (5, 2 )

(b) (2, 5)
(d) ( 2, 5)

72. If x , y and z are in AP and tan 1 x, tan 1 y


and tan 1 z are also in AP, then
(a)
(b)
(c)
(d)

x= y= z
2 x = 3y = 6z
6x = 3y = 2 z
6x = 4y = 3z

34 JEE Main Solved Papers


73. Statement I ( p ~ q) (~ p q) is a fallacy.

78. A multiple choice examination has

Statement II ( p q) (~ q ~ p) is a
tautology.

5 questions. Each question has three


alternative answers of which exactly
one is correct. The probability that a
student will get 4 or more correct
answers just by guessing is

(a) Statement I is true, Statement II is true;


Statement II is a correct explanation for Statement I
(b) Statement I is true, Statement II is true;
Statement II is not a correct explanation for
Statement I
(c) Statement I is true, Statement II is false
(d) Statement I is false, Statement II is true

74. If f( x ) dx = ( x ), then x 5 f( x 3 )dx is equal to


1 3
[ x ( x3 ) x2 ( x3 ) dx] + C
3
1 3
(b)
x ( x3 ) 3 x3 ( x3 ) dx + C
3
1
(c) x3 ( x3 ) x2 ( x3 ) dx + C
3
1
(d) [ x3 ( x3 ) x3 ( x3 ) dx] + C
3
(a)

75.

lim

x0

(a)

(b)

1
2

(c) 1

(d) 2

76. Statement I The value of the integral


/3

/6

dx
is equal to /6 .
1 + tan x

Statement II

a f(x) dx = a f(a + b x) dx

(a) Statement I is true, Statement II is true;


Statement II is a correct explanation for Statement I
(b) Statement I is true, Statement II is true;
Statement II is not a correct explanation for
Statement I
(c) Statement I is true, Statement II is false
(d) Statement I is false, Statement II is true

77. The equation of the circle passing through


the foci of the ellipse
centre at (0, 3) is
(a)
(b)
(c)
(d)

x2
x2
x2
x2

+
+
+
+

y2
y2
y2
y2

6y 7 = 0
6y + 7 = 0
6y 5 = 0
6y + 5 = 0

13
35
10
(d) 5
3
(b)

79. The x-coordinate of the incentre of the


triangle that has the coordinates of
mid-points of its sides as (0, 1), (1, 1)
and (1, 0) is
(a) 2 +

(b) 2 2

(c) 1 +

(d) 1 2

80. The term independent

(1 cos 2 x )(3 + cos x )


is equal to
x tan 4 x

1
4

17
35
11
(c) 5
3
(a)

x 2 y2
+
= 1 and having
16
9

of
expansion
10

x +1
x 1
is

2 /3
x
x 1/ 3 + 1 x x 1/ 2
(a) 4
(c) 210

in
of

(b) 120
(d) 310

81. The area (in square units) bounded by


the

curves

y = x,

2 y x + 3 = 0,

X-axis and lying in the first quadrant is


(a) 9
(c) 18

(b) 36
27
(d)
4

82. Let Tn be the number of all possible


triangles formed by joining vertices of
an n-sided regular polygon. If
Tn + 1 Tn = 10, then the value of n is
(a) 7
(c) 10

(b) 5
(d) 8

83. If z is a complex number of unit

modulus and argument , then arg


1 + z is equal to

1 + z
(a)
(c)

2
(d)

(b)

JEE Main Solved Paper 2013

Statement II If the line,


5
y = mx +
( m 0) is the common tangent,
m
then m satisfies m4 3 m2 + 2 = 0.

84. ABCD is a trapezium such that AB and

CD are parallel and BC CD. If


ADB = , BC = p and CD = q, then AB
is equal to
(a)

( p2 + q 2 ) sin
pcos + q sin

(b)

p2 + q 2 cos
p cos + q sin

(c)
(d)

(a) Statement I is true, Statement II is true;


Statement II is a correct explanation for
Statement I
(b) Statement I is true, Statement II is true;
Statement II is not a correct explanation for
Statement I
(c) Statement I is true, Statement II is false
(d) Statement I is false, Statement II is true

p2 + q 2
p2 cos + q 2 sin
( p2 + q 2 ) sin
( p cos + q sin )2

88. If y = sec (tan 1 x ), then

1 3

1
2
(c) 1

85. If P = 1 3 3 is the adjoint of a 3 3

(a)

2 4 4
matrix A and| A | = 4 , then is equal to

(a) 4
(c) 5

(b) 11
(d) 0

86. The intercepts on X-axis made by


tangents
to
the
curve,
x
y = | t | dt, x R, which are parallel to
0
the line y = 2 x , are equal to
(a) 1
(c) 3

35

(b)
(d)

dy
at x = 1is equal to
dx
1
2
2

tan A
cot A
89. The expression
can be
+
1 cot A 1 tan A
written as
(a) sin A cos A + 1
(c) tan A + cot A

(b) sec A cosec A + 1


(d) sec A + cosec A

90. All the students of a class performed poorly


in Mathematics. The teacher decided to give
grace marks of 10 to each of the students.
Which of the following statistical measures
will not change even after the grace marks
were given?

(b) 2
(d) 4

87. Given A circle, 2 x 2 + 2 y 2 = 5 and a


parabola, y 2 = 4 5 x .
Statement I An equation of a
common tangent to these curves is
y = x + 5.

(a) Mean
(c) Mode

(b) Median
(d) Variance

Answers
1.
11.
21.
31.
41.
51.
61.
71.
81.

(c)
(a)
(d)
(c)
(a)
(a)
(c)
(c)
(a)

2.
12.
22.
32.
42.
52.
62.
72.
82.

(d)
(b)
(b)
(a,b)
(d)
(d)
(c)
(a)
(b)

3.
13.
23.
33.
43.
53.
63.
73.
83.

(d)
(c)
(d)
(c)
(c)
(d)
(c)
(b)
(c)

4.
14.
24.
34.
44.
54.
64.
74.
84.

(b)
(d)
(a)
(d)
(*)
(d)
(c)
(c)
(a)

5.
15.
25.
35.
45.
55.
65.
75.
85.

(b)
(b)
(d)
(a)
(d)
(b)
(c)
(d)
(b)

6.
16.
26.
36.
46.
56.
66.
76.
86.

(c)
(c)
(d)
(a)
(c)
(d)
(d)
(d)
(a)

7.
17.
27.
37.
47.
57.
67.
77.
87.

(b)
(c)
(c)
(c)
(b)
(b)
(c)
(a)
(b)

8.
18.
28.
38.
48.
58.
68.
78.
88.

(b)
(c)
(a)
(c)
(a)
(b)
(b)
(c)
(a)

9.
19.
29.
39.
49.
59.
69.
79.
89.

(a)
(d)
(b)
(a)
(b)
(a)
(b)
(b)
(b)

10.
20.
30.
40.
50.
60.
70.
80.
90.

(c)
(c)
(d)
(b)
(c)
(b)
(a)
(c)
(d)

Solutions
Physics

3. Before collision, the mass is m and after collision

1. / The net force in any direction is equal to the


product of mass and acceleration in that
direction. It is Newtons second law.

In equilibrium, net force on the body


Upward force = Downward force
kx0 + FB = Mg

the mass is m + M.
Maximum energy loss
p2
p2
=

2 m 2 (m + M )

p2
Q KE = 2 m

p2 m
2 m (m + M )
1
m
= mv 2

2
m + M

f = m

m + M

(i)

kx0
FB

Mg

Here, kx0 is restoring force of spring and FB is force


of buoyancy.
As the cylinder is half submersed, so force of
L
buoyancy acting on cylinder = Ag
2
[buoyancy = vg = ( AL)g ]
L
Now, Eq. (i) becomes kx0 + Ag = Mg
2
LAg
Mg
Mg
LA
2

x0 =
=
1

k
k
2M

2. / Whenever a continuous shape is rotating in an

4. From Coulombs law, F =


0 =

1 q1q 2
4 0 R 2
q1q 2

4 FR 2
On substituting the units, we get
C2
[AT]2
as 4 is dimensionless
=
0 =
2
N - m [MLT2 ] [L2 ]
= [M1L3 T4 A 2 ]
Y

5. Initial velocity, V = ($i + 2 $j ) m/s


Magnitude of velocty,
V = (1)2 + (2 )2 = 5 m/s

X
Equation of trajectory of
projectile
gx2
(1+ tan2 )
y = x tan
2u2
Q tan = y = 2 = 2

x 1
2
10( x)

[1+ (2 )2 ]
y= x2
2( 5 )2

uniform magnetic field, an emf is induced


across it. To determine the induced emf, we
take an elementary part, then emf induced in
this elementary part is calculated. Now, for
total induced emf in continuous shape is to be
determined by integration within the limits.

6. Amplitude of damped oscillator

Consider small length of rod dx at a distance x


from axial point of rotation

A = A0 e

2l

dx

x
3l

The emf induced across the rod


e=

3l

2 l (x) Bdx

[(3l ) (2 l ) ]
2
5Bl 2
=
2

= B

= 2x

After 5 s,

10( x2 )
(1 + 4) = 2 x 5 x2
25

0. 9 A0 = A0 e

After 10 more second,


A = A0 e

bt
2m

b( 5 )
2m

(i)

( 15 )
2m
3

5b
A = A0 e 2 m

From Eqs. (i) and (ii),


A = 0.729 A0 = A0
Hence, = 0729
.

(ii)

JEE Main Solved Paper 2013


7. / To make the net potential zero, the connection

10. The figure can be drawn as below

should be with alternate polarities. At the


condition, when potential of each capacitor is
zero, the charges on two capacitor are equal.

B
R

Two capacitors are given below


C1
+

C2
+

120 V

200 V

For potential to be made zero after connection,


the charge of both capacitors are equal.

q1 = q 2
C1V1 = C 2 V2
or
120C1 = 200C 2 3C1 = 5C 2

8. Frequency of sonometer wire


f=

v
1
=
2l 2l

= mass per unit length of wire


A = area of x-section of sonometer wire
d = density of sonometer wire
Also, Youngs modulus of elasticity,
Tl
Q Y = stress = Tl A
Y =
strain l / l
Al

Given,

T Yl
=
A
l

f=

Yl
ld

1
2l

l
= 0.01,
l
d = 7.7 103 kg / m3
l =1.5 m,

Y = 2.2 1011 N / m2
2 103

Hz
7
3
01782
.
103 178.2Hz

After solving, we get f =

9. Mutual inductance of two coils


M=

0 R12 R 22
2(R12 x2 )3 / 2

On substituting the values, we get


0 (2 )(20 102 )2
2 [(02
. )2 + (015
. )2 ]

3 cm

s
ex len
conv

3 mm

A
C

OB = OA = R
AC = OC AC
= R 3 mm
[BA = 3 cm]
By Pythagoras theorem,

(3 cm)2 + (R 3 mm)2 = R 2
(3 cm)2 + R 2 2 R (3 mm) + (3 mm)2 = R 2

R 15 cm

[Q1 mm =101 cm]

Refractive index of material of lens,


8
c 3 10
= =
v 2 108
Here, c = speed of light in vacuum = 3 108 m/s
v = speed of light in material of lens = 2 108 m/s
3
So,
=
2
From lens makers formula,
1
1
1
= ( 1)

f
R1 R 2
Here, R1 = R and R 2 =
1
=0

R2
1 3
1
Thus,
= 1
15
f 2

[for plane surface]

f = 30 cm

11. From conservation of energy,

where x = distance between the centres


Flux through the bigger coil

2 R2R2
M = 0 2 1 12
4 (R x )

M=

Plano

As diameter of plano convex lens is 6 cm.

T
1
T
=
2 l Ad
Q = m = vd = ( Al ) d = Ad

l
l
l

Here,

37

(0.3 102 )2

On solving, we get
M = 9.216 1011 9.2 1011 Wb

Total energy at the planet


= Total enegy at the altitude
GMm
GMm 1
(i)

+ (KE)surface =
+ mv A2
R
3R
2
In its orbit, the necessary centripetal force
provided by gravitational force.
mv A2
GMm
=

(R + 2 R ) (R + 2 R )2
GM
3R
From Eqs. (i) and (ii), we get
5 GMm
(KE)surface =
6 R

v A2 =

(ii)

38 JEE Main Solved Papers


12. Electronic circuit of diode detector

Signal

The time constant,


= 2 . 5 107 1012 s = 2.5 105 s
The higher frequency which can be detected with
tolerable distortion is
1
1
f=
=
Hz
2 ma RC 2p 0.6 2.5 105
100 104
4
104 Hz = 10.61 kHz
Hz =
25 1.2
1.2

13. This condition is obtained by applying the


condition that rate of decay of capacitor voltage
must be equal or less than the rate of decay
modulated singal voltage for proper detection of
modulated signal.
Relation between intensities is
I0

(I0/2)

45

= 106.66 V
V1 V2 = 10.04 V

15. The thermodynamic process is given as


2p0
p0

2T0

2T0

T0
V0

2V0

H = nC V (2T0 T0 ) + nC p + (4T0 T0 )

2
14. As, P = V

R
where, P = power dissipates in the circuit,
V = applied voltage,
R = net resistance of the circuit
120 120
R=
= 240 [resistance of bulb]
60
240

For monoatomic gas,


3R
5R
and C p =
2
2
RT0
5R

H=n
+ n

2T
2 0
2
3
= nT0 + 5 nRT0
2
13
13
=
nRT0 =
pV
2
2 0 0
16. The rotating hoop is given below
CV =

240

48

48
120
54

Heat supplied, H = nC V T + nC p T
where, C p and C V are two heat capacities.

According to Malus law,


I
I
1 I
IR = 0 cos 2 (45 ) = 0 = 0
2 2 4
2

60 W

V2 =

IR

(Unpolarised)

V 2 120 120
=
R
240
[resistance of heater]

R = 60
So, from figure,
240
V1 =
120
246
[Q V = IR]
= 117.073 V
V
120

i2 =
=
R2
54
R=

= RC = 100 103 250 1012 s

Req = 240 + 6 = 246


V
120
[before connecting heater]
i1 =
=
Req 246

60

Rough

120 V

120 V

54

120 V

120 V

From conservation of angular momentum,


v
mr 20 = mvr + mr 2
r
r

v= 0
2
Q w = v , rotational changes to translational

JEE Main Solved Paper 2013

39

17. / When piston is slightly displaced through

23. As is increased, there will be a value of above

equilibrium position, it will oscillate under a


net restoring force. The position will perform
SHM with some time period and frequency.

which photoelectron will be cease to come out. So,


photocurrent will be zero.

Pressure applied by piston,


Mg
= p0
A
Mg = P0 A

is required for higher frequency.

25. When radius is decrease by dr.


(i)
x

x0

Let piston be moved by a distance x.


Here, the system is completely isolated to the
pressure, so will be adiabatic.
p0 V0 = pV
[equation of adiabatic process]
p0 Ax0 = pA ( x0 x)
p0 X 0
p=
( x0 x)
The equation of motion for the piston,
p0 x0
A = Frestoring
Mg

( x0 x)

x0
= Frestoring
p0 A 1

( x0 x)
[for very small value of x, x0 x x0 ]
p0 Ax
F=
= 2 x = (2 f )2 x
x0

f=

1 p0 A
1 p0 A
=
2 x0 M
2
MV0

Q x = V0
0
A

18. According to Newtons cooling law, option (c) is


correct answer.

24. For same value of current, higher value of voltage

Q d = k(T T )
2
1
dt

19. Statement I is false and Statement II is true.


20. For charging of capacitor,

q = CV (1 et / )
At t = 2, q = CV (1 e 2 )

21. It will be concentric circles.


22. As, E = B c
where E= electric field,
B = magnetic field
c = speed of EM wave
On putting the values of electric field
|E| = |B||c|= 20 109 3 108 = 6 V/m

Decrease in surface energy


= Heat required in vapourisation (i)
Decrease in surface energy
=T A
[for two surfaces]
= 2T 4 rdr
Heat required in vapourisation
= Latent heat
= ML = V L
= (4 r 2dr ) L
Now from Eq. (i),
2T 4 r dr = 4 r 2L
2T
r=

26. As energy gap, E = h


Here, h is Plancks constant.
1
E
1
, , =
=k
2
Frequency
2
h

(
)
n
1
n

k 2n
= 2

n (n 1)2
1

3
n

27. We know that, the angle of deviation depends upon


the angle of incidence. If we determine
experimentally,
the
angles
of
deviation
corresponding to different angles of incidence and
then plot i (on X-axis) and (on Y-axis), we get a
curve as shown in figure.
Y

i1

i2

Clearly, if angle of incidence is gradually increased


from a small value, the angle of deviation first
decreases becomes minimum for a particular
angle of incidence and then begins to increase as
shown in diagram.

40 JEE Main Solved Papers


28. / The force on the massive charge by two other

30. Consider the small length of rod dx at distance x

charges can be resolved into two components.


The component of electrostatic force along x
will cancel each other while vertical component
of forces to be added to make a net force in that
direction.

from the point O. Determine potential due to this


small part and integrating it in the limit L to 2L.
L
O

The situation is shown below


dV =

y
q/2
F

F
q

V=
x

F sin
2 F cos

Net force in negative y-direction,


Fnet = 2 F cos
Fnet =

Fnet =

( y + a )
2

2 2

y + a2
2

Fnet y

29. The resultant magnetic induction at O.


Bnet = B1 + B2 + BH
N
BH
B1
B2

S
N

S
N

(M1 + M 2 )
= 0
+ BH
4
r3
QB
0 M
equatorial = 4 r 3
=

dV

Q
k dx
2L L
kdQ
V=
=
L
L
x
x
2L 1
Q
dx = Q [log x]2 L
=
e
L
4 0 L
4 0 L L x
Q
=
[loge 2 L loge L]
4 0 L
Q
2L
Q
log
=
=
ln(2 )
4 o L e L 4 0 L

Chemistry

Optical isomerism is exhibited by only those


complexes which lacks elements of symmetry.
[Co(NH3 )3 Cl 3 ] shows facial as well as meridional
isomerism. But both the forms contain plane of
symmetry. Thus, this complex does not exhibit
optical isomerism.

32. C 2 (6 + 6 = 12 )

Bnet

2L

exhibit optical isomerism.


[Here, M = central atom, A = unidentate ligand,
AA = bidentate ligand]

2 3/ 2

kq 2 y
a3

Fnet =

Q V = kQ
r

kdQ
x

31. / Complexes of the type [M A2 ( AA )2 ], [M ( AA )3 ]

(y + a )

dx

q
2 kq y
2
2

2L

F sin

q
2 kq
2

107 (1.2 + 1)
+ 3.6 105
(0.1)3

= 2.56 104 Wb / m2

*1s 2 , 2 s 2 ,
* 2 s 2 , 2 p2 2 p2
= 1s 2 ,
x
y
Since, all the electrons are paired, it is a
diamagnetic species.
*1s 2, 2 s
N2 (7 + 7 = 14) = 1s 2 ,
*2 s 2 , 2 p2 2 p2 , 2 p2

It is also a diamagnetic specie because of the


absence of unpaired electrons.
O 2 (8 + 8 = 16)

or
S2 = 1s 2 , 1s 2 , 2 s 2 , 2 s 2 ,
2 pz2 , 2 p2x 2 py2
* 2 p1
* 2 p1

Due to the presence of two unpaired electrons, O 2


and S2 both are paramagnetic molecules.

JEE Main Solved Paper 2013


SbCl 5

33. Cl CH CH3 Ph CH CH3 + SbCl 6

Toluene
Carbocation

(planar)
Ph
Due to planar structure of carbocation, both
retention and inversion can take place.

H
Cl
Ph
Cl

C = Average speed =

C = Mean square speed corrected as rms =

CH3

is highest,

hence MnO 4

NOTE As no option correspond to mean square speed, it


is understood as misprint. It should be root mean square
speed.

39. / Find E a by putting values of other quantities in


Arrhenius equation
Ea 1
k
1

log 2 =

.
2.303 R T2 T1
k1

is the

strongest oxidising agent.

35. From first law of thermodynamics, E = q + W for


an isothermal expansion.
Hence,
q = W
q = + 208 J
W = 208 J

36.

Mf =

[expansion work]

M1V1 + M2V2
V1 + V2

750 0.5 + 250 2


750 + 250
875
=
1000
=

OH
>

37.

OH
>

NO2

Cl

(M,I )

(I)

>

CH3

1 1

T2 T1
k1
r
= 1
k2 2 r1

(i)

On putting values in Eq. (i), we get


Ea
1 1
log 2 =
2.303 8.314 310 300

OH

(+I,
hyper
conjugation)

From Arrhenius equation,


k
Ea
log 2 =
k1 2.303 R
k1
r
= 1

k2 r2
k1 1
=
k2 2
k2

=2
k1
Given, T2 = 310 K; T1 = 300 K

= 0.875 M

OH

3RT
M

4
3
:

2
= 1 : 1.128 : 1.225

34. Higher the SRP, better is oxidising agent. Among


the given

8RT
M

*
C < C< C
*
C : C : C = 1:

Therefore, a recemic mixture of 1-chloro-1- phenyl


ethane will be obtained.
E 2+
MnO 4 /Mn

38. C* = Most probable speed = 2RT

Cl

C+

41

OCH3
(+M)

Electron releasing group decreases while electron


withdrawing group increases acidic strength by
destabilising and stabilising the phenoxide ion
formed respectively.
The presence of an electron withdrawing group
increases the acidic strength by stablising the
phenoxide ion while the presence of electron
releasing group destabilises the phenoxide ion
and decrease the acidic strength.

Ea = 53598.6 J / mol = 53.6 kJ / mol

40. R NH2 + CH3 C Cl


O

R NH C CH3
( HCl)

Since, each COCH3 group displaces one H


O

atom in the reaction of one mole of CH3 C Cl


with one NH2 group, the molecular mass
increases with 42 units. Since, the mass increases
by (390 180) = 210, hence the number of NH2
210
group is
= 5.
42

42 JEE Main Solved Papers


41. V 2+ = 3 unpaired electrons; Cr 2+ = 4 unpaired electrons
2+

Mn

= 5 unpaired electrons; Fe

2+

= 4 unpaired electrons

Hence, the order of paramagnetic behaviour should be


V 2 + < Cr 2 + = Fe 2 + < Mn2 +
(b) Ionic size decreases from left to right in the same
period.
(c) (As per data from NCERT)
Co 3+ / Co 2+ = 1.97; Fe 3+ / Fe 2+ = 0.77;

(d) The oxidation states increases as we go from group 3 to


group 7 in the same period.

> CH2 CH CH2 > CH3 CH2 CH2

43. / Make the two half reactions and balance them by any
one of the methods ion electron method or oxidation
number methods.

The half equations of the reaction are


MnO 4 Mn2+

C 2O 24 2CO 2 + 2e

like
and

48. 6CO 2 + 12NADP . 2H + 18 ATP


C 6H12O 6 +18 P + 12NADP + 18 ADP

from left to right and decreases down a


group. The position of given elements in
the periodic table is as
2 16 18

On adding both the equations, we get


2MnO 4 + 5C 2O 4 + 16H+ 2Mn2+
+ 2 5CO 2 +

16
HO
2 2

Thus, x, y and z are 2, 5 and 16, respectively.

Ca

Ar

Ba Se
Thus, the order of increasing HIE1 is
Ba < Ca < Se < S < Ar

44. (a) ONCl = 8 + 7 + 17 = 32 e

(correct)

O
O
2

state (Network like structure,


diamond). Iodine, phosphorus
sulphur are molecular solids.

50. Ionisation energy increases along a period

On equating number of electrons, we get


2MnO 4 + 16H+ + 10 e 2Mn2+ + 8H2O

So, both H2+


2

the charge on oppositely charged ion,


greater is its coagulating power. Since
arsenic sulphide is a negatively charged
sol, thus the order of coagulating power is
Na + < Ba 2+ < Al 3+ .

The balanced half equations are


MnO 4 + 8H+ + 5e Mn2+ + 4H2O

(b)

Nb Na 2 2
=
=0
2
2
and He 2 do not exist.

49. According to Hardy Schulze rule, greater

CO 2

ONO = 8 + 7 + 8 + 1 = 24 e

order do not exist.


0
H2+
2 (1 + 1 2 = 0) = 1s , Bond order = 0
* 1s 2
He (2 + 2 = 4) = 1s 2 ,

47. Silicon exists as covalent crystal in solid

Propyl
(stabilised by
+I effect)

10CO 2 + 10 e

= 0.07 : 0.08 = 7 : 8

Bond order =

CH2

5C 2O 24

0.84 0.08
:
12
1

42. The order of stability of carbocation will be

C 2O 24

C : H=

46. Species having zero or negative bond

Sc 3+ is highly stable (It does not show + 2).

Allyl
(resonance
stabilised)

0.72 g H2O contains 0.08 g H.


44 g CO 2 contains 12 g C.
3.08 g CO 2 contains 0.84 g C.

Empirical formula = C 7H8

Cr 3+ / Cr 2+ = 0.41

Benzyl
(more resonance
stabilised)

45. 18 g H2O contains 2g H.

Central O atom is sp hybridised with 1 lone pair, so


bent shape (correct).
(c) In solid state, ozone is violet-black (correct).
(d) O 3 has no unpaired electrons, so diamagnetic (correct).

51. E = 2.178 1018 12 12 = hc

1
2
1
1
2.178 1018 2 2
1 2
6.62 1034 3.0 108

1.21 107 m
=

JEE Main Solved Paper 2013


52. Compound A gives a precipitate with alcoholic
AgNO 3 , (here, white is misprinting because the
colour of ppt is light yellow) so it must contains Br
in side chain. On oxidation, it gives C 8H6O 4 , which
shows the presence of two alkyl chains attached
directly with the benzene nucleus.
Since, compound B gives anhydride on heating,
the two alkyl substituent must occupy adjacent
(1, 2 ) position. Thus, A must be

CH2Br

and the reactions are as follows

CH3

CH2 OR

Alcoholic
AgNO3

+ AgBr
CH3

Oxidation

Light
yellow
ppt.

O
COOH

COOH
O
Phthalic acid

Phthalic anhydride

53. SRP value normally increases from left to right in


the period of d-block elements. Some SRP value
are exceptionally higher due to stability of product
ion. e.g., E 3+ 2+ = + 1.57 V;
Mn

/Mn

Co 3+ /Co 2 +

Thus, E 3+
M

/M 2+

= + 1.97 V

is highest for Co.

CH3CH2COONH4 CH3 CH2 C NH2


(C )

( B)
Br , KOH

CH3 CH2 NH2

Hofmann
bromamide reaction

57. / (i) Calculate the bond order. Higher the bond

[H ] = 10 = 0.1 M

pH = 2
[H+ ] = 102 = 0.01 M

Li 2 (3 + 3 = 6) = 1s 2 , *1s 2 , 2 s 2
N Na 4 2
Bond order = b
=
=1
2
2
+
2 * 2
Li 2 (3 + 3 1 = 5) = 1s , 1s , 2 s1
32 1
Bond order =
= = 0.5
2
2
Li 2 (3 + 3 + 1 = 7 ) = 1s 2 , * 1s 2 , 2 s 2, * 2 s1
43 1
Bond order =
= = 0.5
2
2
Stability order is Li 2 > Li +2 > Li 2 (because Li 2 has
more number of electrons in anti-bonding orbitals
which destabilises the species)

58. The reaction of alcohol with Lucas reagent is


mostly SN1 reaction and the rate of reaction is
directly proportional to the stability of carbocation
formed in the reaction. Since, 3 R OH forms
3 carbocation (most stable), hence it will react
fastest.
gas leaked from the storage tank of the union
carbide plant in Bhopal gas tragedy.

60. / Balance the total charge and get the percentage


of M 3 + ion.

For dilution of HCl,


M1V1 = M 2 V2
0.1 1 = 0.01 V2
V2 = 10 L
Volume of water to be added = 10 1 = 9 L

55. Na Na + + e

( A)

59. Methyl isocyanate CH3 N == C == O (MIC gas)

54. pH = 1
+

NH 3
56. CH3CH2 C OH

order, higher is the stability.


(ii) Lesser the number of antibonding
electrons, higher is the stability.

CH3
CH2Br

43

(first IE)

Na + + e Na
Since reaction is reverse, so electron gain enthalpy
of Na + ,
H (e g ) = 5.1 eV

Consider one mole of the oxide.


Moles of M = 0.98 moles of O 2 = 1
Let moles of M 3+ = x
Moles of M 2 + = 0.98 x
On balancing charge, we get
(0. 98 x) 2 + 3 x 2 = 0

1. 96 2 x + 3 x 2 = 0

x = 0. 04
0. 04
100 = 4. 08%
Percentage of M 3 + =
0. 98

44 JEE Main Solved Papers

Mathematics

65. We know that, the sum of three vectors of a triangle

61. Given planes are


2x + y + 2z 8 = 0
5
and
2x + y + 2z + = 0
2
Distance between two parallel planes
5
21
8
| d1 d 2 |
7
2
=
= 2 =
=
3
2
2
2
2
2
2
2
a + b +c
2 +1 +2

is zero.

AB + BC + CA = 0

BC = AC AB
AC AB
(i)
BM =

2
[QBC = 2BM, M is a mid-point of BC]
Also, in ABM, AB + BM + MA = 0
[by properties of a triangle]

62. Given, dP = (100 12 x )


dx

AB +

dP = (100 12 x ) dx

On integrating both sides, we get

dP = (100 12

AC AB
= AM
2
[from Eq. (i), BM =

AC AB
]
2

x ) dx

P = 100 x 8 x3 / 2 + C
When x = 0, then P = 2000 C = 2000
Now, when x = 25, then
P = 100 25 8 (25)3 / 2 + 2000
= 2500 8 125 + 2000

= 4500 1000 = 3500

63. Given, n( A) = 2, n(B) = 4

n( A B) = 8
The number of subsets of A B having 3 or more
elements = 8C 3 + 8C 4 ++ 8C 8
= 2 8 8C 0 8C1 8C 2
= 256 1 8 28 = 219

64. Condition for two lines are coplanar.


[in vector form]
[a c, b, d ] = 0
where, a and c are the points on a line and b, d are
the direction ratios of the lines.
2 1 3 4 4 5
1
1
k =0

k
2
1

1
1
k

1 1
1 k =0
2
1

1 (1 + 2 k ) + 1 (1 + k 2 ) 1 (2 k ) = 0

k2 + 2 k + k = 0

k 2 + 3k = 0

| AM | =

way of representing the equation of straight line is


x2 y3
; z=4
=
1
1

42 + 12 + 42 =

33

66. Let f( x) = 2 x3 + 3 x + k
On differentiating w.r.t. x, we get
f ( x) = 6 x2 + 3 > 0, x R
f( x) is strictly increasing function.
f( x) = 0 has only one real root, so two roots are
not possible.

67. Let S = 0.7 + 0.77 + 0.777 + upto 20 terms


7
77
777
+
+
+ upto 20 terms
10 102
103
1
11
111
= 7 + 2 + 3 ++ upto 20 terms
10 10

10
7 9
99
999
=
+
+
++ upto 20 terms
9 10 100 1000

k = 0, 3

NOTE If 0 appears in the denominator, then the correct

AB + AC
AM =
2
3$i + 4k$ + 5$i 2 $j + 4k$
=
2
= 4$i $j + 4k$

7
9

1 1 + 1 1 + 1 1


10
102
103

+upto 20 terms]
7
+
+
(
1
1
upto
20
terms
)
[
9
1
1
1
+ 2 + 3 + upto 20 terms
10 10

10

JEE Main Solved Paper 2013

7
1
20
9
9

7
=
[179 +
81
=

For no solution,| A | = 0 and (adj A) B 0


k+1
8
Now,
| A| =
=0
k
k+ 3

20
20

1 7 179 1 1
1 10 = 9 9 + 9 10

(10) 20 ]

k 2 + 4k + 3 8k = 0

k 2 4k + 3 = 0

(k 1) (k 3) = 0

k = 1, k = 3
k+ 3 8
adj A =
k k + 1
k + 3 8 4
k
Now, (adj A) B =
k k + 1 3 k 1

1
1

(k + 1) (k + 3) 8k = 0

Now,

Alo
2
n
x + g the
3
l
B (0, 1)
y = ine 1
3
(3, 0)
A
1

68. Take any point B (0, 1) on given line.

X 2

r
ted
ec
l
f
Re

ay

(k + 3) (4k ) 8 (3k 1)
=
2
4k + (k + 1) (3k 1)
X

4k 2 12 k + 8
=

2
k + 2k 1

B (0, 1)

Put k = 1

4 12 + 8 0
, not true
=
(adj A) B =
1 + 2 1 0

Equation of AB
y0=

B = 4 k
3 k 1

and

20

1
1
1

10
10
7

= 20

9
1
1

10

45

1 0
(x
0 3

3y = x +
3y = 3
3y = x

Alternate Solution
Given equation of line
x + 3y =

3
3

(i)

y = 1
Slope of incident ray is

Put k = 3
3)

x
3

1
.
3

So, slope of reflected ray must be

1
and the point
3

of incident ( 3, 0).
So, equation of reflected ray
1
y 0 =
( x 3)
3

3y = x 3

69. Given equations can be written in matrix form.


AX = B
8
k+1

, X = x
where, A =
y
k + 3
k

36 36 + 8 8
0, true
=
(adj A) B =
9 + 6 1 4
Hence, required value of k is 3.
Alternate Solution
Condition for the system of equations has no
solution,
a1
b
c
= 1 1
a2 b2 c 2
k+1
8
4k
=

k
k + 3 3k 1
k+1
8
Take
=
k
k+ 3

k 2 + 4k + 3 = 8k

k 2 4k + 3 = 0

(k 1) (k 3) = 0
k = 1, 3
8
41
, false
=
k = 1, then
1+ 3
2

If

And if k = 3 , then

8
4 3
, true

6 91

Therefore, k = 3
Hence, only one value of k exists.

46 JEE Main Solved Papers


73. Statement II ( p q ) (~ q ~ p)

70. Given equations are


and

x2 + 2 x + 3 = 0

(i)

ax2 + bx + c = 0

(ii)

Since, Eq. (i) has imaginary roots, so Eq. (ii) will


also have both roots same as Eq. (i).
a b c
Thus,
= =
1 2 3
Hence, a : b : c is 1 : 2 : 3.

71. Let the centre be (3, k ) and radius r.


Clearly, equation of circle be
( x 3)2 + ( y k )2 = r 2

(i)

QCircle passes through (3, 0), therefore we have


[using Eq. (i)]
(3 3)2 + (0 k )2 = r 2

k2 = r2 k = r
[Qcentre lies in IVth quadrant]
Now, Eq. (i) become ( x 3)2 + ( y + r )2 = r 2
Y
A (3,0)
X

O
2

(1,2)

(3, k)

Q Circle also passes through (1, 2 ), therefore we


have
4 + (r 2 )2 = r 2

4 + r 2 + 4 4r = r 2 r = 2

k = 2
Equation of circle is
( x 3)2 + ( y + 2 )2 = 4

( x 3)2 + y2 + 4 + 4 y = 4

( x 3)2 + y2 + 4 y = 0

72. / If a, b, c are in AP, then 2b = a + c and if d, e, f


are in GP, then e2 = df .

Since, x, y and z are in AP.

2y = x + z
Also, tan 1 x, tan 1 y and tan 1 z are in AP.

2 tan 1 y = tan 1 x + tan 1( z)

2y
1 x + z
tan 1
= tan

2
1 xz
1 y
x+ z
x+ z
y2 = xz
=

1 y2 1 xz
Since x, y and z are in AP as well as in GP.

x= y= z

(p q) (p q)
which is always true, so Statement II is true.
Statement I ( p ~ q ) (~ p q )
p ~q ~ p q
p ~ p ~q q f f f
Hence, it is a fallacy Statement.
So, Statement I is true.
Alternate Solution
Statement II ( p q ) (~ q ~ p)
Since, ~ q ~ p is contrapositive of p q ,
therefore ( p q ) ( p q )
Hence, ( p q ) ( p q ) will be a tautology.
Statement I ( p ~ q ) (~ p q )
~p ~q

p ~q ~p q

( p ~ q)
(~ p q)

Hence, it is a fallacy.

74. Given, f( x) dx = ( x)
I=

Let

f( x3 ) dx

x3 = t x2dx =

Put

dt
3

(i)

1
1
t f(t ) dt = [ x3 ( x3 ) 3 x2 ( x3 ) dx] + C
3
3
[from Eq. (i)]
1
= x3 ( x3 ) x2 ( x3 ) dx + C
3

I=

75. / To solve problem, use cos 2x = 1 2 sin 2 x , then


take the limit.

Let I = lim

x 0

(1 cos 2 x) (3 + cos x)
x

tan4 x
1
x2

2 sin2 x 3 + cos x
x

1
tan 4 x
x2
2
sin x
4x
= 2 lim
lim (3 + cos x) lim
x 0 4 tan 4 x
x 0 x
x 0
1
= 2 4 = 2
4
/3
dx
(i)
76. Let
I=
/6 1+
tan x
/3
dx
I=

/6

1 + tan x
2

= lim

x 0

JEE Main Solved Paper 2013

I=

/3

tan x dx
tan x

/ 6 1 +

(ii)

/3

79. Given, mid-points of sides of a triangle are (0, 1,)


(1, 1) and (1, 0). Plotting these points on a graph
paper and make a triangle.

On adding Eqs. (i) and (ii), we get


2I =

/3

/ 6 dx 2 I = [ x] / 6

2 3 6 12
Statement I is false.
I=

C (0,2)
2

But f( x) dx =

by property of definite integrals.


2

16

B
(0, 0)

77. Given equation of ellipse is x + y = 1.

ax1 + bx2 + cx3


,
a+ b+c
ay1 + by2 + cy3
a+ b+c
where, a, b, c are sides of the triangle]
[Qincentre of circle

X
Y

16

7
, 0
Foci is ( ae, 0) = 4
4

7
4

Here, a = 4, b = 3, e = 1

x-coordinate of incentre
2 2 + 2 2 0 + 2 0
2 2
2
=
=

2+2+2 2
2+ 2 2 2
=2 2

7 ,0)

Radius of the circle, r = (ae ) + b


2

80. / Here,

firstly
use
the
formula
a 3 + b3 = (a + b) (a 2 ab + b2), then apply the
Binomial theorem for general term in the
expansion of (x + a) n .

= 7+ 9
= 16 = 4

Tr +1= nC r xn r ar

Now, equation of circle is


( x 0)2 + ( y 3)2 = 16

A (2, 0)

So, the sides of a triangle will be 2, 2 and 2 2 + 2 2


i .e.,2 2.

x+1
( x 1)
2/3

1/ 3
1/ 2
x x + 1 x x

x + y 6y 7 = 0
2

(1, 0)
2

= (

(1, 1)

2 (0, 1)

a f(a + b x) dx is a true statement,

47

78. / To solve above problem, use Binomial

10

( x1/ 3 )3 + 13
{( x )2 1}

= 2/ 3

1/ 3
x ( x 1)
x +1
x

distribution of probability
n
Cr p r qn r, r = 0, 1, 2, ... , n

Probability of guessing a correct answer, p =

1
3

and probability of guessing a wrong answer,


2
q=
3
Here,
n = 5, r = 0, 1, 2, 3, 4, 5
The probability of guessing 4 or more correct
answers
4

1
2
1
= 5C 4 + 5C 5
3 3
3

[Q using Binomial distribution n C r pr q n r ]


2
1
11
= 5 5 + 5 = 5
3
3
3

10

= ( x1/ 3 x 1/ 2 )10
The general term is
Tr

+1

C r ( x1/ 3 )10 r ( x 1/ 2 )r

10

10

C r ( 1)r x

10 r
r

3
2

For independent of x, put


10 r r
=0
3
2

20 2 r 3r = 0

20 = 5r r = 4
10! 10 9 8 7
10
= 210
T5 = C 4 =
=

6! 4! 4 3 2 1

48 JEE Main Solved Papers


81. Given curves are y = x

(i)

2y x + 3 = 0

and

84. Let AB = x

x
y

=
+3

O
1 1 2 3
2 0, 3
2
3
4
Y

(+)

y = x

A
9

( x 3) ( x + 1) = 0

x=3

p
xq
p
tan ( + ) =
q x

q x = p cot ( + )

x = q p cot ( + )
cot cot 1
= q p

cot + cot

q cot p
= q p

q + p cot

y=3
Required area =
=

0 (line - curve ) dy
3

q cos p sin
= q p

q sin + p cos

{(2 y + 3) y2 } dy
3

y3
= y2 + 3 y
3

0
=9+ 99=9

82. Given,

n+1

Tn + 1 Tn =

n+1

C 3 nC 3 = 10

C 2 + C 3 C 3 = 10
[Q nC r + nC r
n

But

[given]

n=5
z = e i
1
z

1+ z
arg
= arg ( z) =
1 + 1

Cr ]

p2
+q

n+1

C 2 = 5C 2

z=

( p2 + q 2 ) sin
p cos + q sin

AB =

Alternate Solution
Applying sine rule in ABD,

C3

83. Given,| z| = 1, arg z =

q 2 sin + pq cos pq cos + p2 sin


p cos + q sin

C 2 = 10

x=

Tn = nC 3
Tn + 1 =

Qcot = q
p

q cot 1

=q p p

q
+ cot

[Q x = 1is not possible]

In DAM, tan ( ) =

2 x ( x )2 + 3 = 0
( x) 2 x 3 = 0

On solving Eqs. (i) and (ii), we get

p2
+q

xq

4
3
1
1
X

(ii)

(+)
B

p2 + q 2
p2 + q 2
AB
AB
=
=

sin sin { ( + )}
sin sin ( + )

AB =

p2 + q 2 sin
sin cos + cos sin

( p2 + q 2 ) sin
Qcos =
q sin + p cos

2
2
( p + q ) sin
=
Qsin =
p cos + q sin

p + q

2
2
p +q
q

JEE Main Solved Paper 2013


85. Given,

1 3
P = 1 3 3
2 4 4
| P | = 112
( 12 ) (4 6) + 3 (4 6)
= 2 6
[given]
P = adj( A)
|P| = |adj A| =| A|3 1 = | A|2 = 16
[Q|adj A| = | A|n 1, order is 3 3]

m4 + m2 2 = 0

(m2 + 2 ) (m2 1) = 0

m= 1

88. Given, y = sec (tan 1 x)


1+

|t| dt , parallel to y = 2 x

Points, y =

Let

y 2 = 2 ( x 2)

x = tan

y = sec = 1 + x2

At x = 1,

y + 2 = 2 ( x + 2)

tan A
cot A
+
1 cot A 1 tan A
sin A
sin A
=

cos A sin A cos A


cos A
cos A
+

sin A cos A sin A

0 2 = 2 ( x 2)
0 + 2 = 2 ( x + 2)
x=1

87. / The equation of tangent of slope m to circle


x 2 + y2 = a 2

is

y = mx a 1 + m 2

and

equation of tangent of slope m to parabola


a
y 2 = 4ax is y = mx + . With the help of these
m
equations, find the equation of common
tangent.

Equation of circle can be rewritten as


5
x 2 + y2 =
2
Let common tangent be
5
y = mx +
[m 0]
m
The perpendicular from centre to the tangent is
equal to radius.
5
5
m

=
2
2
1+ m

m 1 + m2 =

m2 (1 + m2 ) = 2

dy
1
=
dx
2

89. Given expression is

For x-intercept, put y = 0, we get


and

tan 1 x =

On differentiating w.r.t. x, we get


dy
1
=
2 x
dx 2 1 + x2

| t | dt = 2

Equation of tangent is
and

dy
= | x| = 2
dx
x=2

86. Given, tangents of the curve

[Qm2 + 2 0, as m R ]

y = ( x + 5 ), both statements are


correct as m = 1 satisfies the given equation of
Statement II.

2 6 = 16
2 = 22
= 11
y=

49

sin3 A cos 3 A
1
sin A cos A cos A sin A

sin2 A + sin A cos A + cos 2 A


sin A cos A

1 + sin A cos A
sin A cos A

= 1 + sec A cosec A

90. If initially marks of students were xi , then


( xi x )2
N
Now, each is increased by 10.
12 =

22 =

[( xi + 10) ( x + 10)]2
= 12
N

So, variance will not change whereas mean,


median and mode will increase by 10.

You might also like